If K delivers the third speech, any of the following could be the student who makes the fourth speech EXCEPT

yckim2180 on June 9, 2021

Please help

Could you please provide an explanation on how to arrive at A?

Reply
Create a free account to read and take part in forum discussions.

Already have an account? log in

Emil-Kunkin on December 1 at 03:31PM

K being third doesn't seem to give us much information here, so I would immediately try out the answer choice that tells us the most. The only answer we really know much about is h, since h is before s, and putting hk together tells us about j. So I'd test h out first.

_ _ K H _ _

We know that s would be one of the last two, and we know that j cannot be 2 or. 5 as this would violate the first rule. However this would force j into either one or six, would violate violate the j rule, so this cannot happen.